OBF 2017- Segunda Fase (Nível 2)

Solução de Victor Ivo

Questão 01 (Exclusiva pro primeiro ano)-

a) A primeira coisa a ser observar é que a aceleração do bloco na subida é diferente da aceleração do bloco da subida, disto temos que:

F=ma=mg sen{\alpha}+\mu N

N=mg cos{\alpha}

a=g(sen(\alpha)+\mu cos(\alpha))

O atrito está no sentido contrário ao movimento, e neste caso está na mesma direção da componente tangencial do peso ( que decompomos em normal e tangencial ao longo do plano), aplicando torriceli:

v^{2}=v_{1}^2-2ad

Onde d é o que é percorrido por ele, estando ele parado no final:

v^{2}=0=v_{1}^2-2ad

d=\frac{v_{1}^2}{2g(sen(\alpha)+\mu cos(\alpha))}

Aplicando torriceli na descida, a partir do ponto em que ele para:

v_{2}^2=0+2a_{descida} d

E a aceleração da descida é, agora que a gravidade e atrito estão em direções contrárias ( por que a partícula está descendo o plano):

F=ma=mg sen(\alpha)-\mu mg cos(\alpha)

a=g(sen(\alpha)-\mu cos(\alpha))

b)

Daí:

v_{2}^2=2ad=2g(sen(\alpha)-\mu cos(\alpha) \frac{v_{1}^2}{2g(sen(\alpha)+\mu cos(\alpha)}

v_{2}^2=v_{1}^2\frac{sen(\alpha)-\mu cos(\alpha))}{sen(\alpha)+\mu cos(\alpha)}

v_{2}^2(sen(\alpha)+\mu cos(\alpha))=v_{1}^2(sen(\alpha)-\mu cos(\alpha)

\mu cos(\alpha) (v_{2}^2+v_{1}^2)=(v_{1}^2-v_{2}^2) sen(\alpha)

\mu=tg(\alpha) \frac{v_{1}^2-v_{2}^2}{v_{2}^2+v_{1}^2}

Daí, substituindo na equação para d:

d=\frac{v_{1}^2}{2g(sen(\alpha)+\mu cos(\alpha))}

d=\frac{v_{1}^2}{2g(sen(\alpha)+\frac{v_{1}^2-v_{2}^2}{v_{1}^2+v_{2}^2} tg(\alpha) cos(\alpha))}

d=\frac{v_{1}^2}{2g sen(\alpha) \frac{2v_{1}^2}{v_{1}^2+v_{2}^2}}

d=\frac{v_{1}^2+v_{2}^2}{4g sen(\alpha)}

Gab: a)

\mu= tg(\alpha)\frac{v_{1}^2-v_{2}^2}{v_{1}^2+v_{2}^2}=\frac{7\sqrt{3}}{75} \approx 0.16 \approx 0.2

Para ter um significativo, como as velocidades, você apenas deixaria 0.2, apesar de apenas ter uma casa, é o certo devido às poucas casas da veloocidade.

b)  

d=\frac{v_{1}^2+v_{2}^2}{4g sen(\alpha)}=1.25 m \approx 1 m

Para ter um significativo, como as velocidades, você apenas deixaria 1 m, apesar de bem tosco, é o certo teoricamente.

Questão 02-

Podemos interpretar que a energia líquida transferida ao sistema se refere à parte do trabalho que não se tornou calor, e assim o calor não é transferido ao cubo e vai pra superfície ou que leva muito tempo para se propagar pelo cubo. Desta maneira, podemos achar a energia que foi pro cubo menor por proporção de volumes, primeiro, a energia líquida mecânica do sistema é:

K=W-Q=800 J

\frac{K}{V_{tot}}=\frac{k}{V_{menor}}

k=K\frac{V_{menor}}{V_{total}}

k=K(\frac{a}{l})^3

Onde a é o lado do cubo menor e l o do maior, logo, substituindo:

k=800 (\frac{0.5}{1})^3=1*10^{2}J

Gab:

k=1*10^{2} J

Questão 03-

Podemos encontrar isso por conservação de energia, que implica conservação do volume do líquido de trabalho, pois o trabalho realizado sobre o primeiro pistão é:

W=p A_{1} \Delta h_{1}

Que é compensado pelo trabalho do pistão 2:

W=p A_{2} \Delta h_{2}

A_{2} \Delta h_{2}= A_{1} \Delta h_{1}

\Delta h_{2}=\Delta h_{1} \frac{A_{1}}{A_{2}}

\Delta h_{2}=2*\frac{100}{400}=0.5 cm

Gab:

\Delta h_{2}=0.5 cm

Questão 04-

Podemos usar aqui que o trabalho de um ciclo é numericamente igual à sua área delimitada no diagrama PV, i.e, apenas temos que calcular quanto vale a área dessas circunferências concêntricas. Também precisamos notar que não podemos pegar o raio da circunferência como sendo algo medidando apenas "atm" ou "l". Dessa maneira podemos ignorar o fato de que temos uma circunferência no ciclo e podemos calcular essa área como a área de uma elipse, que dará a dimensão correta para a conta.

S=\pi ab

Sendo a e b os semi eixos maior e menor, respectivamente, que seria o "raio" da nossa circunferência, mas que varia dependendo do eixo que você pega. Perceba também que cada raio corresponde a um quarto de "circunferência", e que se denotarmos as circunferências de 1 a 4 no sentido anti horário, sendo 1 a inferior esquerda, os raios estão na proporção 1:2:3:4, logo:

W=\frac{\pi a_{1} b_{1}}{4} (1+2^{2}+3^{2}+4^{2})

Pelo gráfico:

a_{1}b_{1}=1l * 1atm=100 J

Logo:

W=750 \pi \approx 2.25* 10^{3} \approx 2.3 * 10^{3}

Deixamos apenas dois dígitos na resposta final porque o gráfico é graduado em 1, nos dando pelo menos um número apos a vírgula para os valores de cada parâmetros, nos deixando com dois significativos.

Gab:

W \approx 2.25 * 10^{3} \approx 2.3* 10^{3}

Questão 05-

Podemos aplicar o que sabemos de gravitação universal para encontrar o período de um sistema binário com órbita circular, dado que o centro da circunferência está entre os corpos e que a linha que liga eles sempre passa pelo centro de massa e que os planetas giram com a mesma frequência, que é necessário para o movimento, temos pela segunda lei de Newton e força gravitacional:

F_{1}=R_{centrip}=m_{1}\omega^{2} r_{1}=\frac{Gm_{1}m_{2}}{(r_{1}+r_{2})^2}

\omega^{2} r_{1}=\frac{Gm_{2}}{(r_{1}+r_{2})^2}

Em que r_{1} é o raio de circunferência do primeiro corpo celeste, e r^{2} o segundo, sendo portanto a distância entre eles simplesmente a soma, que chamaremos posteriormente de r. Fazendo a equação análoga para a segunda massa e somando as duas, temos:

\omega^{2} (r_{1}+r_{2})=\frac{G(m_{1}+m_{2})}{(r_{1}+r_{2})^2}

\omega^{2}=\frac{G(m_{1}+m_{2})}{r^3}

Usando a relação do período com a frequência angular T=\frac{2\pi}{\omega}, temos:

\frac{r^{3}}{T^{2}}=\frac{G(m_{1}+m_{2})}{4\pi^2}

Portanto, disso podemos perceber que o enunciado tem informação desnecessário, pois a partir da informação dada sobre a razão poderíamos encontrar rapidamente o valor da massa a partir desta equação, então o tempo que a luz leva pra ir de um corpo a outro é desnecessário. Contudo, a partir da relação dada na prova:

m_{lua}=\frac{4\pi^2}{G(1+\frac{m_{terra}}{m_{lua}})} \frac{r^3}{T^2}

Substituindo os valores:

m_{lua}\approx\frac{4*9}{6.7*10^{-11}*82} 1.1*10^{3} \approx 7.2*10^{12} kg

Que é um valor com ordem de grandeza 10 vezes maior que o correto da lua!! O que nos levar a pensar que não seria isso o que a questão queria que você encontrasse, principalmente pelo fato de existir informação a mais na questão. Apesar de tudo isso, trabalhar com os dados da questão como sendo apenas a constante gravitacional, valor de tempo para um raio de luz ir de planeta a outro e razão entre raio ao cubo pra período ao quadrado só deixar essa massa como resposta, sendo essa, portanto, a correta pelos dados da questão.

Gab:

m_{lua} \approx 7.2*10^{12} kg

Questão 06-

A força de empuxo que a água faz sobre o corpo é igual e contrária à força que o corpo faz sobre a água devido ao empuxo, esta que é equilibrada pela superfície embaixo do corpo, podemos escrever a equação de equilíbrio do sistema como:

N=P_{recipiente}+P_{corpo}-T

Onde N é a normal da balança (que é o que efetivamente a balança registra como peso/massa medida), T é a tração da corda e P's são os pesos respectivos, logo, aplicando o equilíbrio ao corpo também:

P_{corpo}=E+T

E=N-P_{recipiente}

E=P_{efetivo}-P_{recipiente}=(m_{medida}-m_{recipiente})g

E=1.3 N

Gab:

E=1.3 N

Questão 7 (Exclusiva para primeiro ano) -

Basta usarmos que o tempo que o caminhão leva para chegar ao ponto da colisão é igual ao tempo que o objeto leva para cair, usando o que conhecemos das equações da cinemática:

\Delta x=V_{caminhao} t

\Delta h=\frac{gt^{2}}{2}

Onde apenas usamos que o caminhão se move a velocidade constante e o corpo cai em queda livre sem velocidade inicial, daí, igualando os tempos:

\frac{\Delta x}{V}=\sqrt{\frac{2\Delta h}{g}}

\frac{L}{V}=\sqrt{\frac{2L}{2g}}

V=\sqrt{gL}

Gab:

V=\sqrt{gL}

Questão 08 (Exclusiva para primeiro ano)-

Sabemos da teoria do cone de Mach que eles se formam quando temos objetos se propagando mais rápido que a velocidade do som no meio, ou em geral mais rápido que a velocidade das frentes de onda, e trabalhando com a definição dele, temos que para manter o formato de cone deve valer que o tanto que uma onda anda para chegar do ponto de onde foi lançada até o ponto em que vai estar na frente de onda num instante depois deve ter uma certa relação com o tanto que o objeto andou, podemos tirar isso da trigonometria do cone.

Cone de Mach

Figura 1: Frentes de onda geradas pela fonte para diferentes tipo de velocidade do objeto, perceba que são séries de círculos cada vez menos concêntricos que formam um lugar geométrico de intersecções com uma reta fazendo um certo angulo com a velocidade do objeto. v é a velocidade do objeto e v_{s} é a do som ou frente de onda genérica.

A partir desse ângulo \phi, perceba que tomando um ponto de intersecção genérico de uma circunferência, que chamaremos de A, e traçando perpendiculamente até a linha horizontal, sendo o ponto de contato com ela o ponto B, o tamanho desse traçado, AB deve ser v_{s} \Delta t pois é o caminho percorrido nesse tempo por uma onda do meio. Também note que esse traçado é a hipotenusa de um triângulo retângulo de hipotenusa formado pelo caminho do corpo, sendo este o triângulo ABC, o ponto C é a posição atual do objeto, a hipotenusa BC deve ser de tamanho v\Delta t pois é o caminho traçado pelo objeto de velocidade v.

 sen(\phi)=\frac{AB}{BC}=\frac{v_{s}\Delta t}{v \Delta t}

sen(\phi)=\frac{v_{s}}{v}

Para esse ângulo \phi existir a velocidade do corpo deve ser maior ou igual ao da onda, e no caso de igualdade o cone na verdade é uma parede vertical, perpendicular à velocidade do objeto, percebendo pela nossa imagem que o ângulo \phi é metade de 60^{\circ}.

sen(30^{\circ})=\frac{1}{2}=\frac{4}{v}

v=8 \frac{m}{s}

Gab:

v=8 \frac{m}{s}

 

Questão 09-

a) Se considerarmos que vale as condições de Gauss e que o tempo de propagação da luz é muito rápido, podemos aplicar a equação dos pontos conjugados:

\frac{1}{f}=\frac{1}{p}+\frac{1}{p'}

Da figura vemos que o ponto está a 10 cm do centro de curvatura, portanto p=20cm+10cm=30cm:

\frac{1}{10}=\frac{1}{30}+\frac{1}{p'}

\frac{1}{p'}=\frac{1}{15}

p'=15cm

b) A partir dos pontos conjugados podemos encontrar a fórmula da velocidade instantânea da imagem, pois sabemos que:

\frac{d}{dt}(\frac{1}{x})=-\frac{1}{x^2}\frac{dx}{dt}

Daí, da equação de gauss, sendo o foco do espelho constante no tempo:

\frac{d}{dt} (\frac{1}{f})=-\frac{1}{f^2}\frac{df}{dt}=0

\frac{d}{dt} (\frac{1}{p})+\frac{d}{dt} (\frac{1}{p'})=0

-\frac{1}{p^2}\frac{dp}{dt}-\frac{1}{p'^2}\frac{dp'}{dt}=0

\frac{dp'}{dt}=-(\frac{p'}{p})^2 \frac{dp}{dt}

Como o objeto está se aproximando do vértice a velocidade constante, a distância p diminui a uma taxa constante no tempo, que supondo uma velocidade de 10 \frac{cm}{s} (o primeiro dado da questão está provavelmente errado pela ordem de magnitude, e fora isso a velocidade dada durante a passa é realmente em cm, então é o que deveríamos considerar mesmo):

\frac{dp'}{dt}=V_{imagem}=-(\frac{15}{30})^2 (-10) \frac{cm}{s}

\frac{dp'}{dt}=V_{imagem}=2.5 \frac{cm}{s}

E o sinal positivo indica que a imagem está se afastando do vértice, se considerarmos que os dados todos tem apenas 2 algarismos significativos, seria mais adequado aproximar:

V_{imagem}=2.5 \frac{cm}{s}

E seria mais adequado ainda transformar as grandezas pro SI, que faremos no gabarito.

 Gab: a)

p'=15cm

            b)

V_{imagem}=2.5 \frac{cm}{s}= 0.025 \frac{m}{s}

Questão 10-

Para fazer esta questão usaremos que a velocidade do barco em relação ao rio é constante, e que portanto a taxa de consumo do combustível é constante, que é o que faz mais sentido físico já que o que faz o combustível funcionar é a necessidade do barco se mover em relação a alguma coisa, neste caso o rio. Como a potência do motor é constante, o gasto de combustível é proporcional ao tempo, logo:

G=Pt

Onde G é o gasto que sabemos e P é a potência, que é desconhecida. O tempo gasto para atravessar o rio na direção ao longo da corrente é:

t_{rio}=\frac{L}{v_{b}+v}

E na direção contrária da corrente:

t_{anti-rio}=\frac{L}{v_{b}-v}

Divindo os gastos, temos:

\frac{G_{rio}}{G_{anti-rio}}=\frac{t_{rio}}{t_{anti-rio}}=\frac{v_{b}-v}{v_{b}+v}

v_{b}=v\frac{G_{rio}+G_{anti-rio}}{G_{anti-rio}-G_{rio}}

Substituindo:

v_{b}=1.8 \frac{9+6}{9-6}=9 \frac{km}{h}

E deixamos apenas um algarismo no 9 porque o menor número de algarismos significativos está nos litros gastos, e é um, lembremos também que temos que colocar as respostas no SI, portanto:

v_{b}=9 \frac{km}{h} \frac{\frac{m}{s}}{\frac{3.6 km}{h}}=2.5 \frac{m}{s} \approx 3 \frac{m}{s}

Onde tivemos que fazer uma aproximação um pouco feia, mas que teve de ser feita porque a resposta deveria ter o número correto de significativos, recomendo colocar a resposta exata e depois a trabalha por algarismos sempre na prova pra evitar qualquer confusão.

Gab:

 v_{b}= 2.5 \frac{m}{s} \approx 3 \frac{m}{s}

Questão 11-

Existem uma deflexão do raio, na direção vertical, devido à passagem da luz num meio mais refrigente, pois a angulação do raio com a horizontal será diferente neste meio, podemos inclusive encontrar isto por lei de snell, no meio, sendo a lámina vertical, tem o ângulo da luz com a sua normal como sendo o ângulo da luz com a horizontal, logo:

 n sen(\alpha)=sen(30^{\circ})

sen(\alpha)=\frac{1}{3}

E disso achamos o cosseno e a nova tangente:

cos^2{\alpha}+sen^2{\alpha}=1

cos(\alpha)=\frac{2\sqrt{2}}{3}

tg(\alpha)=\frac{\sqrt{2}}{4}

O caminho vertical feita pelo raio de luz na lâmina é:

\Delta y=tg(\alpha) * \Delta x=2 *\frac{\sqrt{2}}{4}=\frac{\sqrt{2}}{2} cm

O deslocamento vertical da luz caso não tivesse a lâmina seria:

\Delta y'=tg(30^{circ}) \Delta x=\frac{1}{\sqrt{3}} 2=\frac{2\sqrt{3}}{3} cm

\Delta y'-\Delta y=\frac{2\sqrt{3}}{3}-\frac{sqrt{2}}{2}

O raio real se desloca menos que o raio original:

d=\frac{2\sqrt{3}}{3}-\frac{\sqrt{2}}{2}

d=\frac{4\sqrt{3}-3\sqrt{2}}{6} cm

E como o raio real desloca menos, temos que ele vai atingir acima do raio prolongado, então temos que abaixar o rifle de uma distância d do original.

Gab:

Você deve abaixar o rifle de uma distância:

d=\frac{4\sqrt{3}-3\sqrt{2}}{6}cm \approx 0.43 cm \approx 0.4 cm

Colocamos o resultado de no final como 0.4cm para ter apenas um significativo, como o número com menos significativos do problema (2 cm de espessura).

Questão 12-

Podemos partir da equação conhecida do período duma mola para obter a resposta, mas para sermos mais claros na solução pra quem não conhecia a equação antes iremos demosntrar a equação a partir da definição de M.H.S, falamos que uma equação descreve um M.H.S de frequência angular (pois descrevemos um M.H.S com uma fase) \omega como o que respeita a equação de movimento:

a+\omega^{2} x=0

Sendo a a aceleração do corpo na direção de x, usando a definição de força elástica de uma mola a partir da lei de hooke:

F_{el}=-kx

Num sistema descrito como o anterior onde temos, numa maneira geral, n molas, com uma i-ésima de constante elástica k_{i}, e tendo a massa usada um certo peso:

F_{resul}=mg-k_{1}x-k_{2}x-...-k_{n}x

F_{resul}=mg-\sum_{i=1}^n k_{i} x

F_{resul}=- \sum_{i=1}^n k_{i} (x-\frac{mg}{\sum_{i=1}^n k_{i}})

Definindo a variável x' como:

x'=x-\frac{mg}{\sum_{i=1}^n k_{i}}

Derivando a equação duas vezes (se você não sabe cálculo, imagine que estamos pegando a aceleração por meio das duas coordenadas, e que como uma difere da outra apenas a partir de uma constante, a aceleração é a mesma)

\frac{d^2 x'}{dt^2}=a'=\frac{d^2 x}{dt^2}-\frac{d^2}{dt^2} (\frac{mg}{\sum_{i=1}^n k_{i}})

a'=\frac{d^2 x}{dt^2}=a

Agora com a segunda lei de Newton:

F_{resul}=ma=ma'=-(\sum_{i=1}^n k_{i}) x'

a'+\frac{\sum_{i=1}^n k_{i}}{m} x'=0

Então o movimento resultante disso será um M.H.S de frequência angular e período:

\omega=\sqrt{\frac{\sum_{i=1}^n k_{i}}{m}}

T=\frac{2\pi}{\omega}=2\pi \sqrt{\frac{m}{\sum_{i=1}^n k_{i}}}

Perceba que a frequência do movimento não depende do peso da massa, e em geral isso vale, uma força constante sobre o sistema não muda o período do M.H.S, então você poderia ter resolvido o problema ignorando a presença do peso. Agora aplicando nossa equação geral aos casos particulas, sendo T_{1} correspondente ao caso de uma mola e T_{2} de duas:

T_{1}=2\pi \sqrt{\frac{m}{k}}

T_{2}=2\pi \sqrt{\frac{m}{k+k}}=2\pi \sqrt{\frac{m}{2k}}

T_{2}=\frac{T_{1}}{\sqrt{2}}

Substituindo os valores:

T_{2}=\frac{1.4}{\sqrt{2}} \approx 1.0 s

Gab:

T_{2} \approx 1.0 s